ADV ORTHO FINAL

Réussis tes devoirs et examens dès maintenant avec Quizwiz!

2. You have been treating a 35 y/o electrician for subacromial impingement for the last 6 weeks. His main deficits at this point are in scapular stabilizer strength and rotator cuff weakness. He is pain-free today in your session. You want an exercise to best target his infraspinatus and teres minor. The most appropriate exercise of the choices listed below is: Shrugs with 20# weights in each hand Theraband shoulder external rotation at 90 degrees of abduction Theraband shoulder internal rotation at 0 degrees of abduction Sidelying shoulder external rotation with 1#

. Answer B is correct because it is targeting the infraspinatus and teres minor, and it is in a functional position allowing the patient to train the muscles in a position he will be in often for work.

2) You are working with a collegiate athlete post OP ACLR and she is anxious to return to playing field hockey. So far in therapy she has been doing very well and has progressed quickly through her ROM and strengthening exercises. She wants to be cleared to return to sport, but what initial assessments must you make to ensure her safety before you even begin return to sport testing? Quad strength symmetry, pain free 30min walk, joint effusion Hamstring symmetry, pain free ROM, patellar movement Knee pain, pain free ROM, active hyperextension, joint effusion Knee varus / valgus, pain free SLS, patellar movement

: C is correct because these are the assessments that show the pt.'s knee has healed enough to safely perform the return to sport tests. A, B, and D are incorrect because those observations are not as valid for pt. clearance to participate in return to sport testing post ACLR.

Pt is a 17 y/o female reporting to PT s/p R ACL construction 6 weeks ago. Which of the following is the most important goal for the patient at this stage of rehab? A. Improving cardio endurance B. Increasing endurance and strength of R quad C. Improving R knee extension to be equal to L D. Balance and SLS on R

: C is the best answer to this question. Knee extension is the harder part of ROM to get back after knee surgery/ injury and therefore should be the first priority to ensure proper range is retuned before the knee scars down. A, B, and D are also important during this stage especially getting the quad to fire and strengthen again due to the fact that it can shut down and atrophy quickly after surgery due to non-use and swelling in the area.

Which of the following is not an appropriate intervention for the acute stage of shoulder rehabilitation? A. Self stretch of posterior capsule B. Submaximal isometric IR and ER C. Two handed chest ball pass D. Rhythmic stabilization in supine at 90 degrees of flexion

: C would not be a good choice for a patient in the acute stage of rehabilitation as it requires more control, strength and proper form that is likely lacking at this stage and could cause more damage. A, B and D would all be appropriate for this stage and important for early shoulder mobility, muscle activation, and shoulder stabilization.

3. It has been a four weeks since John first came into your clinic and after performing a reassessment you find that he no longer has strength deficits, however he states that he occasionally has increased fatigue at the end of long games. What is your next plan for John's treatment? a. Incorporate neuromuscular control drills such as perturbations at end range and sport-specific positions b. Continue to perform shoulder IR and ER strengthening with the TheraBand c. Discharge John from PT because he does not have any strength impairments d. Refer John back to his physician

A is correct. When working with baseball pitchers/overhead athletes it is important to address proprioception. It is common to for someone to experience increased symptoms/pain with fatigue secondary to decreased proprioception. Incorporating sport specific and neuromuscular control into John's treatment is essential. B is not the best answer because his strength is 5/5, however it is important to note that you are testing John's strength at his "best" and that he may test differently when fatigued, therefore B is incorrect. It is not advised to discharge John from therapy because he continues to experience symptoms, therefore C is incorrect. Finally, D is incorrect because there are no red flags, John has improved but continues to have aspects to work on and his pain can be reproduced.

You are evaluating a 40 year old male who presents with Achilles tendinopathy. Which of the following is the most important approach to strengthening? a. Concentric strengthening of the quadriceps and hamstrings b. Eccentric strengthening of the glutes c. Eccentric strengthening of the gastroc, soleus, and posterior tibialis d. Concentric strengthening of gastroc, soleus, and posterior tibialis

A major contributing factor to the onset of Achilles tendinopathy is increased pronation. There are many reasons that excessive pronation occurs, but lack of control from muscles such as the posterior tibialis and gastroc/soleus complex is one that we can easily help change as physical therapists. Eccentric strengthening of these muscles will help to control excessive pronation. Studies show that this type of strengthening for patients with Achilles tendinopathies decreases overall pain by over 50%. Therefore, the correct answer is C.

3. Mike is a 17yo high school athlete who underwent shoulder surgery. He is 3 weeks post op and today is his first therapy session since the initial evaluation. Which of the following would not be appropriate interventions for today's session? a. Side lying scaption b. Submaximal isometric strengthening c. Supine manual pec minor stretch d. 90-90 doorway stretch

A would not be appropriate for this patient to perform today because side lying scaption requires AROM. During the acute phase of rehab patients are performing ROM exercises that are PROM working towards AAROM. B, C and D are appropriate during this phase to allow for restoration of normal ROM and preventing muscle atrophy. These interventions will not be harmful to the surgical site. Along with these interventions the focus should be continuing to decrease pain and inflammation allowing for optimal healing.

You are working with a 20-year-old college baseball pitcher s/p SLAP lesion repair. Which of the following interventions would not be appropriate during the acute rehabilitation phase? a. TENS and ice b. PROM and AAROM of the shoulder c. Prone scapular muscle strengthening d. Thrower's Ten Program

A, B, and C are all appropriate interventions for the acute rehab phase. The goal of the initial phase of rehab is to minimize pain and swelling (A), restore normal motion (B), and begin to restore normal muscle balance between the anterior and posterior muscles of the shoulder (C). The posterior/scapular muscles tend to be weak in most people, especially in throwers. Strengthening the periscapular muscles will help to create a more stable shoulder. Answer D is appropriate further along in the rehab process when considering the patient's return to sport.

2. Carli is a 16 yo soccer player who you have been treating since her ACL reconstruction surgery 6 months ago. Carli is about ready for discharge but you are hesitant for her to go back to soccer because her motor control of the knee is not great. You decide to test her motor control with three tests before making a decision. Which of the following is not a test for motor control? a. Lateral leap and catch b. T- Test c. Drop jump d. Timed lateral step down

A, C and D are all motor control tests that can be done at this time. These tests help to look at the motor control happening at the knee. With decreased control and increased knee valgus during these tests it may indicate that the patient still needs to improve strength and control before returning to sport. B. is testing the agility of the patient and would not be as important in providing information about motor control.

2. 16 yo F patient is 6 months s/p R ACLR. Patient demonstrates full, pain free knee ROM, active hyperextension with superior patellar glide, 1/10 knee pain, and trace amounts of joint effusion. What is the most appropriate next step of treatment? a. Initiate return to sport functional testing. b. Discharge and clear athlete to return to sport. c. Continue with current rehab protocol without progression. d. Regress rehab program due to slight reports of pain

A. Patient has passed all 4 clearance parameters to begin functional testing for return to sport. B is incorrect because it is not appropriate to clear a patient for return to sport without administering return to sport tests. Answer C and D are incorrect because the patient's presentation indicates readiness for progression of treatment.

3. All of the following are characteristics of strength training EXCEPT: a. 60-80% of 1RM b. 80-100% of 1RM c. 6-12 reps d. 3-5 minute rest period

A. Represents the amount of load required for endurance training. B,C and D represent strength training because need high loads, moderate reps with a moderate rest period. Endurance training requires lighter loads with higher reps and lower rest periods. And for power training it requires moderate loads, low reps and longer rest periods, however it is important to go through the reps quickly as a component of power is speed.

2. An increased step rate by 5% during running is associated with which of the following: A) greater vertical displacement B) less energy absorbed at the knee C) greater foot inclination angle D) heel landing farther from COM

Answer B: According to the study by Heiderscheit et al. on the effects of step rate on running mechanics: increasing the step rate by 5% for a runner led to an average of 20% less energy absorbed at the knee.

3. Which of the following is least associated with patellofemoral syndrome? A) Foot pronation B) Increased femoral inclination C) Increased knee flexion angle D) Decreased hip abduction strength

Answer C is correct because according to studies by Crossley et al (2004) and Boling et al (2009) decreased knee flexion angle (rather than increased) is an associated risk factor for developing patellofemoral syndrome. A decreased knee flexion angle results in less contact area of the patella.

Robert comes into your outpatient clinic s/p R meniscus repair 8 weeks ago. Robert reports that he has been feeling better, but still has increased soreness after long days at high school. What is the most appropriate PT intervention for Robert today? a. As a result of Roberts reports of soreness, do not progress him to CKC load/strengthening. b. Continue improving Robert's R knee ROM, however do not exceed 90 degrees of flexion. c. Progress to full pain-free ROM d. Robert should not be sore 8 weeks after surgery, advise him/his mother to call the surgeon immediately.

Answer C. Robert is 8 weeks s/p R meniscus repair, therefore he is in the intermediate post-operative phase of rehabilitation. It would be appropriate to begin a CKC progression, therefore A is incorrect. Also, B is incorrect because at this point Robert should be working towards regaining full ROM, keeping in mind that there are no further restrictions from the surgeon and complex factors to challenge rehab. C is correct because Robert should progress to regaining full ROM, however it is important to note that this should be pain-free. It is essential to monitor Roberts response. Finally, D is incorrect because reports of soreness is not a red flag to immediately call the surgeon. It will be important for Robert to monitor his signs/symptoms as his activity levels and treatment progress.

. While watching a patient run on a treadmill from behind, you notice that she demonstrates excessive vertical displacement that looks like a "bounce" while running. What is the least likely cause of this? A) hip flexor tightness B) Gastroc tightness C) Decreased talocrural mobility D) Midfoot hypermobility

Answer D: Midfoot hypermobility Midfoot hypermobility is more likely associated with over-compliance, which will cause the patient to demonstrate a lateral pelvic tilt, hip IR and adduction, knee valgus, and foot pronation. Answers A, B, and C can all contribute to excessive vertical displacement. The similarity between these answers are restrictions in either the anterior or posterior chain.

Robert continues to see you at 12 weeks s/p R meniscus repair. Robert reports no pain/soreness with walking and reports no increases swelling at the end of the day or after physical therapy treatments. What in an appropriate intervention for Robert at this time? a. Perform 3 sets of 10 repetitions of squats to a 90 degree angle. b. SLS on airex pad 3x30 seconds with internal and external perturbations. c. Gradually introduce biking during treatments. d. A and B are correct e. B and C are correct f. A, B, C are all correct

Answer E. A is incorrect because Robert should not exceed 70 degrees until ~16 weeks. B is correct because it is appropriate for Robert to improve dynamic stability/balance. C is correct because it is appropriate for Robert to gradually introduce biking/elliptical/swimming, however it is essential to monitor Roberts s/s and subjective reports. As stated above A is incorrect, therefore D and F are also incorrect.

Your 35 y/o male patient comes in with leg pain during running. You decide to analyze his running pattern on the treadmill and find he has excessive crossover when he runs. This may be due to which of the following? A. Adductor tightness or Posterolateral hip weakness B. Hip flexor tightness or ITB tightness C. Gastroc/soleus tightness or talocrural hypomobility D. Midfoot hypermobility or poor motor control

Answer: A, Posterolateral hip weakness (glute med weakness) and adductor tightness are causes for crossover during running. When this occurs, the foot crosses over the midline, there is lateral pelvic tilt, pronation, and toe out/foot eversion. B is incorrect hip flexor tightness is a cause for excessive vertical displacement during running, and ITB tightness is not related to cross over during running. C is also incorrect because gastroc/soleus tightness or talocrural hypomobility are also causes for excessive vertical displacement. D is incorrect because midfoot hypermobility/poor motor control are reasons for over-compliance.

2. A 45 y/o overweight female pt comes in for R foot pain. She says her pain is worst when she first wakes up in the morning but feels better after she walks around. Upon evaluation you find her R DF is 0 degrees and she pain on palpation on medical calcaneal tubercle. She does not c/o burning/tingling/numbness in the R foot. What might you suspect as a differential diagnosis for this patient? A. Tarsal tunnel syndrome B. Medial tibial stress syndrome C. Plantar fasciopathy D. Achilles Tendinopathy

Answer: C, Patient presentation for plantar fasciopathy includes pain on palpation medial calcaneal tubercle, 1st step pain in AM, decreased pain with movement, limited ankle dorsiflexion, and increased BMI with nonathletic individuals (Martin RL et al. JOSPT. 2014). This pt presents with these symptoms and risk factors such as high BMI and decreased DF. A would be incorrect because this patient does not complain of any neurological symptoms such as numbness/tingling/burning in the foot. B would be incorrect because patients with medial tibial stress syndrome present with pain on palpation over distal posterior medial tibia, while the patient in this question presents with pain on palpation over medial calcaneal tubercle. D would be incorrect because patients with Achilles tendinopathy typically present with pain along distal 1/3 of tendon, and have pain with passive dorsiflexion.

This same patient, who has had a two-tunnel transtibial root repair done two weeks ago, comes for a physical therapy session. Which of the following would be a goal for treatment for this patient? a. Restore extension ROM/patellar mobility b. Minimize muscular atrophy/restore quadriceps control c. Initiate hamstring strengthening d. A&B e. B&C

Answer: D (A&B). This answer is correct because choices a&b are both appropriate goals in the early post-operative phase (0-6 weeks). Restoring extension ROM and patellar mobility is an appropriate goal, as you want to avoid contractures and flexion contractures in particular. Minimizing muscular atrophy and restoring quadriceps control is another appropriate goal as you want to strengthen the leg muscles as to avoid atrophy and muscle imbalances. Answer C is incorrect as hamstring strengthening should not be initiated until the intermediate post-operative phase (weeks 6-12), and week 2 would be too early to begin hamstring strengthening. Answer E is incorrect as well because although answer B is correct, it includes answer C which is incorrect.

John is a 20 year old division I football player. He is seeing you today after following R ACL reconstruction 6 months ago. John states that he ruptured his R ACL while celebrating a winning touchdown. He reports that there were no complications with his surgery and that he feels "ready" to get back with the team. While observing John walking into your clinic you can already see prevalent gait abnormalities and minimal effusion, before performing any objective measurements. After completing your examination with John it will be important to educate him on... a. About 82% of people who have a primary ACLR return to sport and approximately 62% of athletes return to their pre-injury level with Level 1 sports. Explaining to him that there is a balance between time to get back to football, his performance, and injury prevention. b. Secondary to his age and mechanism of injury (non-impact) he has an increased risk of re-injury to both is affected knee and contralateral knee. It will be important to focus on R>L and bilateral lower extremity rehabilitation; directing your focus on structural and function abnormalities and why John had an increased risk on tearing his ACL in the first place. c. It has been six months since John's R ACLR and he is ready to get back to football practices and games. The gait, strength, and higher level abnormalities seen during your examination will improve with practice and time. d. Both A and B are correct.

Answer: D is correct because you will want to educate John on both options A and B. It will be important to educate John as much as you can about his risk factors to injury/re-injury of his ACL. As well as, educating him on the importance of true ready to return to sport and the potential consequences if he returns to football earlier. It is also important to note that John is probably seeing many professionals, such as his physician, surgeon, athletic trainer, coach etc. If these professionals are encouraging John to play sooner than what you recommend, he should have all of the education and tools to make an educated decision for himself. Option C is incorrect because determining readiness to return to sport is not based on a time criteria. It is likely that John's abnormalities will increase and/or begin to affect negatively affect his overall health, performance, etc.

3. Your 23 year old female patient c/o pain in the distal portion of the Achilles tendon and has pain with passive dorsiflexion and resisted plantarflexion. She is an avid runner who typically trains 50+ miles a week. Upon examination you find her glute strength is 4-/5 bilaterally, plantarflexion strength is 4-/5 bilaterally, she has limited flexibility in her hip flexors and gastroc-soleus, limited dorsiflexion to neutral, and she has normal joint mobility in STJ. Which of the following would NOT be appropriate for a patient with Achilles tendinopathy? A. Hip flexor stretching B. Gastroc-Soleus stretching C. Eccentric calf strengthening D. Subtalar joint mobilizations to increase dorsiflexion

Answer: D. This would be an inappropriate intervention for this patient because she has normal joint mobility in STJ, so her limited DF is most likely due to her gastroc-soleus tightness. A is incorrect because this patient has hip flexor tightness so she would benefit from hip flexor stretching. B is incorrect because this patient has limited DF and gastroc-soleus tightness, so she would benefit from gastroc-soleus stretching. C would be incorrect because eccentric calf strengthening are effective for patients with Achilles tendinopathy to address faulty biomechanics, and this patient has decreased plantarflexion strength

Which one of these is false regarding closed kinetic chain movements? A) Joint mechanoreceptors are maximally stimulated with compression B) CKC movements are detrimental for healing and repaired tissues C) CKC enhances stability and dynamic control D) Many sports involve CKC movements

B is FALSE. Due to compressive load, shear, translation, and distraction are minimized. CKC movements are protective for healing and repaired tissues.

) You are working with a young football player to help him build endurance as a wide receiver. You perform some tests to calculate his 1RM and now it's time to design an appropriate exercise program. What percentage of his 1RM should you be around and around how many reps should you be prescribing? 50% - 80% of 1RM for 0-6 repetitions w/ 5-8 minute rests 60% - 80% of 1RM for 12-20 repetitions w/ 30-60 second rest 80% - 100% of 1RM for 6-12 repetitions w/ 3-5 minute rests 100% of 1RM for 1-2 repetitions w/ breaks as needed

B is correct because lower weight w/ higher repetitions helps to build muscle endurance which is what the wide receiver will need for sport. A is incorrect because moderate weight w/ lower repetitions helps build Power. C is incorrect because high weight w/ lower repetitions helps build strength. D is incorrect because just doing 1RM w/ breaks as needed won't be as effective as a slightly lower weight w/ more repetitions and standard breaks.

You are working with a pt. that just started training for her first marathon a month ago that she is running for charity. She states she's never really been all that active and is following the "couch to marathon" training program she found online. She's at the clinic for pain in her legs that always starts around the 10mi mark during her runs. After her eval you take note that she has a BMI of 25, palpation to the medial leg is tender, and she has navicular drop bilaterally. What do you think her potential diagnosis is? Patellofemoral pain syndrome Medial Tibial Stress syndrome Patella tendonosis Plantar fasciitis

B is correct because of the incr. strain she is placing on her bones, her incr. BMI, gender, decr. previous activity, and predictable onset of pain at 10mi are all predictors of MTSS. A is incorrect because PFPS is more related to mechanical/strength deficiencies and age factors. C and D are incorrect because palpation revealed no knee tendon / plantar pain.

Which is true regarding Glenohumeral Internal Rotation Deficit (GIRD)? a. IR deficity is >10% of total rotation b. Tight anterior capsule c. >100 degrees is dangerous and excessive d. All of the above

B is incorrect because an internal rotation deficit is primarily due to a tight posterior capsule. C is incorrect because >120 degrees of rotation is considered excessive especially if there is poor scapular control. A is correct because >10% deficit of total rotation is concern for poor control. The more arc of rotation that is seen, that is a further range of motion that requires good control of motion.

Which of the following return to sport tests s/p ACL rehab is the best for looking at a patient's motor control? A. Maximum voluntary isometric contraction B. Timed lateral step down C. Vertical Single leg jump D. Y balance test

B is the best answer because although other tests also look at motor control, this test forces the pt to go at the speed of the metronome which can force them to have to move at a challenging speed that may show more motor control issues as they are forced to adapt their speed, motion, and control without relying on their own motor planning. A and C look more at strength and power. The Y balance test also looks at control but since the pt is in control of their own speed and distance they are moving out of BOS it may not challenge motor control as greatly as the timed lateral step down.

. You have been treating a young male volleyball player for shoulder impingement syndrome for about four weeks now. He has made significant gains in scapular muscle strength and motor control, but you notice he continues to present with a moderately protracted shoulder girdle. You have addressed his rotator cuff, scapular retractors, and serratus anterior to decrease scapular winging. What could you address during this visit to further promote an upright posture and scapular retraction with this patient? a. Begin the patient with prone I-T-Y exercise b. Assess pec minor length and address with manual stretching c. Apply tape to the upper thoracic spine of the patient for tactile cuing for upright posture d. Educate the patient on upper thoracic foam rolling for HEP

B is the correct answer because as the question states, the physical therapist had addressed most of the factors that could have been associated with the patient's impingement symptoms. However, it is important to also address pec minor while strengthening serratus anterior or the patient will be limited in their gains. While serratus anterior posteriorly tips the scapula, a tight pec minor could anteriorly tip the scapula which would limit the patient's progression and potentially continue to cause impingement symptoms. Answer A is incorrect because it is addressing the strength component which has already been addressed with this patient. C. Continues to focus on the posterior components of scapular retraction which have been previously addressed with strengthening exercises with significant gains. And D. Along with A and C continues to focus posteriorly and with the strengthening exercises this patient has done so far, posture would be improved.

You just learned about the basic strengthening principles in class. You decide that you want to do supersets with your Garrett, who has knee pain after playing a game of basketball. What is the best choice of exercises to alternate between? A) Knee extension machine and terminal knee extension with theraband B) Knee extension and hamstring curls C) Hamstring curls and calf raises D) Bridges and 4 way ankle band

B is the correct answer. A superset is when you alternate agonist/antagonist with minimal rest. Knee extension exercises would be working the quadriceps, and hamstring curls would be working the hamstrings— therefore, alternating agonist/antagonists. Option A is an example of compound set, as you are would be alternating exercises for the same muscle group.

Patient is a 25 y/o avid runner coming into the clinic with knee pain with running. Upon performing a running assessment it is noted that the patient has excessive vertical displacement. Which of the following is NOT a factor that may be contributing to this running pattern? A. Tightness of gastroc and soleus B. Reduced trunk rotation C. Hyopmobility of the talocrual joint D. Hip flexor tightness

B is the correct answer. Excessive vertical displacement is caused by restrictions in the sagittal plane of motion, while trunk rotation is in the transverse plane. A, C, & D all occur in the frontal plane and these restrictions force a runner to compensate with the "bounce like" pattern. The excessive force upon lading often causes stress to the patellofemoral joint that can lead to pain.

1. Which of the following would be the most appropriate exercise to perform with an overhead athlete in order to increased ER strength to improve the ER:IR strength ratio? a. SL ER with 2 lb weight b. Standing ER with shoulder at 90 degrees with therband resistance c. Isometric ER in doorway d. Prone ER at 90 degrees of ABD

B is the most appropriate answer because the athlete is working in a more functional overhead position to strengthen the musculature. Additionally they are getting resistance into ER and then having to control the theraband as they come back into IR, which is an important aspect of the throwing mechanism. B and C are not in the most functional positions. A is a position that can get the greatest EMG activation but again is not the most functional.

3. 18 yo tennis player presents to clinic c/o R shoulder pain when serving/hitting the ball overhead. ROM measurements are WNL but pain reported with ER/IR above 90o ABD. Strength deficits of rotator cuff muscles, middle and lower trapezius, serratus anterior, latissimus dorsi and teres major, and core musculature. Joint play reveals posterior GH capsule tightness. Which of the following interventions would be the MOST appropriate for the next treatment session? a. Standing scaption with IR. b. Sidelying ER with a towel roll under the arm and manual resistance concentric and eccentrically. c. Standing ER at 90o ABD with resistance band. d. Prone horizontal ABD on physiotherapy ball.

B. This intervention is the most appropriate level and targets muscle groups of noted strength deficits. Answer A is incorrect because it puts the patient in a position of impingement which will likely cause more pain. Answer C is incorrect because it is too advanced of an exercise at this time. Answer D is incorrect because it is too advanced of an exercises at this time due to patient's noted core weakness.

. John is a 20 year old pitcher presenting to you with right shoulder pain. John pitches for his local university and mentioned that his right shoulder pain (dominant arm) has been gradually increasing. Additionally, he states that his accuracy has been decreasing. After performing a detailed subjective history and objective tests and measures you notice that John's right shoulder ER is 7 degrees greater than his left, and his right shoulder IR is 7 degrees less than his left. As a result of John's measurements and his complaints of right shoulder pain you should do which of the following? a. Instruct John to perform a sleepers stretch with his right shoulder to increase right shoulder IR since it is less than his left shoulder. b. Perform a manual ER stretch to John's left shoulder because you want his shoulder ER to be symmetrical since he is a pitcher. c. You know that this finding is not uncommon for baseball pitchers in their dominant arm, however follow up subjective questions and objective measurements are warranted in order to address John's pain and symptoms. d. Refer John back to the physician.

Both A and B are incorrect because you do not want to increase John's ROM. A study done by Wilk & Arrigo JOSPT 1993 found this presentation is common among baseball pitchers when comparing dominant versus non-dominant shoulders. Additionally, if John is having difficulty controlling his shoulder in space while he is pitching, increasing his ROM will increase his difficulty. C is correct because John is not limited in ROM, however we need to mechanically reproduce John's symptoms and gain a better understanding of his injury. D is incorrect because there are no red flags to be concerned with.

You are working with a young high school running athlete who presents with significant cross-over gait pattern when running. What exercises would be BEST to teach this pt. in order to address his cross-over gait and make him more efficient? Prone hamstring curls to strengthen his hamstring ms. Standing heel raises to strengthen his gastroc ms. Lateral walks with a theraband to strengthen his glute med ms. Supine leg lifts to strengthen his hip flexors ms.

C is correct because Glute Med weakness is the main cause for a cross-over gait pattern in the frontal plane of movement. A, B, and D are incorrect because they mainly provide strength and stability in the Sagittal plane of movement during gait.

2. You are treating Stephanie, a soccer player in your outpatient clinic. She tells you she sometimes feels her muscles are fatigued during running. You come to the conclusion that you need to work on muscle endurance to improve her performance. What intensity should you have in mind when making her exercise program? A) Loads between 80%-100% of 1 RM; 6-12 repetitions B) Loads between 50%-80% of 1 RM; 0-6 repetitions C) Loads between 60%-80% of 1 RM; 12-20 repetitions D) Loads between 60%-80% of 1 RM; 0-6 repetitions

C is the correct answer. For endurance training you want to focus on having loads between 60%-80% of 1 RM, 12-20 repetitions, and 30-60 second rest periods. Doing lighter loads and more repetitions will work on endurance. A is incorrect, those are parameters for strength training. B is incorrect, those are parameters for power training- as power training has a component of speed to it. And D is not a formalized parameter for either strength, endurance, or power training.

Pt is a 50 y/o male reporting to PT with B arch pain and pain with palpation the medial calcaneal tubercle. Which of the following would be the least effective treatment for his condition? A. Anti-pronation taping of B feet B. Standing stretch of gastoc and soleus C. Ultrasound to the plantar surface of the feet D. Posterior mobilization of the talocrual joint

C is the correct answer. There is conflicting evidence on the use of modalities to treat plantar fasciopathy. Stretching of the calves has been shown to provide some pain relief. Anti-pronation taping can also help to support the arch and prevent the excessive pronation common in these individual. Finally joint mobilizations have been proven to be effective treatment and the posterior mobilization would help to improve dorsiflexion that is often lacking in this population.

3) Based on the following findings, in what phase of rehabilitation is this particular OH athlete? - Minimal pain/tenderness at shoulder - Full PROM and AROM of affected shoulder - Good strength and endurance - Symmetrical capsule mobility - Decreased proprioception (15 degrees off with proprioceptive positioning tests) A) Acute Phase B) Intermediate Phase C) Advanced Phase D) Return to Activity Phase

C. Clinical Rationale: The patient presents with minimal pain, full ROM and increased strength, so answer A could not be correct. To progress to the intermediate phase, a patient would only require near full PROM and muscle balance of the shoulder complex. The patient has full AROM and PROM, good strength/endurance, symmetrical capsule mobility, and decreased pain which will mean that answer B is incorrect. Answer D would be incorrect because a patient would require adequate proprioception before returning to activity. Therefore, answer C is correct.

3. Which of the following is the most appropriate exercise progression to strengthen shoulder ER in an overhead athlete? Sidelying ER, ER in scapular plane, ER at 0 degrees with towel roll, ER at 90 degrees Sidelying ER, ER at 90 degrees, ER at 0 degrees with towel roll, ER in scapular plane ER at 0 degrees with towel roll, sidelying ER, ER in scapular plane, ER at 90 degrees ER at 0 degrees with towel roll, ER at 90 degrees, sidelying ER, ER in scapular plane

C. Clinical rationale: ER strengthening at 90 degrees ABD is the most functional position for an overhead athlete, however it is not a good place to start due to the increased strain on the shoulder capsule in that position. Instead, begin by strengthening the ER at 0 degrees ABD and progress towards 90 degrees once the athlete has adequate strength to complete the exercise in a safe and controlled manner. ER in sidelying is a progression from ER in standing because gravity adds increased resistance. ER at 90 is a progression from ER in the scapular plane because of the increased ABD. Therefore, answer C is the best choice.

Mike is a 17 yo basketball player you are treating post-op ACL reconstruction using hamstring tendon autograft. His surgery was on 7/1/17 and he us currently 2 weeks post op. Which of the following interventions would not be appropriate for him at this time? a. Quad Sets b. SLR c. Ball squats d. Heel slides

C. would not be appropriate for Mike at this time because he is only 2 weeks post op and still in the early phase of tissue healing. You need to continue to protect the healing tissues at this time. Closed kinetic chain exercises should not be included in treatment until the intermediate stage around 6 weeks. A,B and D are all good interventions to perform with this patient at this time. These exercises are non weight bearing and help to restore ROM, strengthen the quadriceps and hip flexors while protecting the healing tissues.

3. Which of the following would be considered a compound set of exercises? A) Bench press/seated row B) Bench press/tricep extension C) Bicep curls/tricep extension D) Quad set/Hamstring curl

Clinical Rationale: The correct answer is B) Bench press/tricep extension. A compound set refers to alternating exercises for the same muscle group, which in this case would be the triceps. A superset refers to a set of exercises that alternate between agonist and antagonist muscle groups. Answers B,C,D are all considered supersets and therefore cannot be the right answers.

A patient comes into your clinic following an ACL reconstructive surgery for her R knee. You are looking to test the patient's maximum voluntary isometric contraction (MVIC) in order to properly prescribe strengthening exercises. At what position should the patient be tested to determine the patient's MVIC at the quad? A) 80 degrees of knee flexion B) 45 degrees of knee flexion C) 60 degrees of knee flexion D) 0 degrees of knee flexion

Clinical Rationale: The correct answer is C) 60 degrees of knee flexion. An individual's MVIC should be calculated in quad neutral which is is 60 degrees of knee flexion. Answers B and D would not be considered quad neutral as this position causes a posterior glide at the tibia. Answer A is incorrect as it causes an anterior glide at the tibia. Both of these positions would cause strain on the ACL which the patient should be tested in the quad neutral position of 60 degrees.

In the intermediate phase of knee strengthening following a surgical procedure, which of the following would be the most appropriate intervention? Endurance focus: low load, high repetition exercises Strength/hypertrophy focus: high load, low repetition exercises Strength/hypertrophy focus: gradually increase load, moderate repetitions Challenge stabilizing muscles outside of base of support

Clinical Rationale: Answer A is incorrect because that is more consistent with the early phase of knee strengthening, while the goal is still to protect the healing tissues. Answer B (high load, low repetition) is incorrect because that is more consistent with the advanced stages of strengthening following a surgery, and would be too advanced for where the tissue was in this stage of healing. Answer D is incorrect because that is also too advanced for this stage in healing; there is not enough baseline strength in the stabilizing muscles to begin challenging the patient outside of the base of support in more compromising positions. Answer C is correct because it is promoting strengthening of the musculature that helps to support the knee, which will be important to attain first before moving on to more challenging activities. joint.

A PT is trying to determine an athlete's readiness to return to sport following an ACL reconstruction. The PT just observed the athlete perform a drop jump, where the aim of the test was to assess motor control. Which of the following would be considered an abnormal finding on this task during landing? Asymmetrical landing Decreased knee flexion Dynamic knee valgus All of the above

Clinical Rationale: Answers A, B, C would all be considered abnormal findings during a jump task, which makes answer D: all of the above, correct. An athlete that presents with an asymmetrical landing (A) may mean that there is pain, apprehension, or decreased strength on the operated knee. An athlete that presents with decreased knee flexion (B) during landing indicates that they are not allowing for proper shock absorption, and are putting themselves at risk for reinjury by landing on a knee that is too stiff. An athlete that lands with a dynamic knee valgus (C) could indicate that there is decreased strength proximally in the hip external rotators, or that they have not adequately trained to promote better movement patterns when jumping. All of those findings could put an athlete for an increased risk of injury to the operated knee, and possibly the healthy knee as well.

1) You are working with a young pitcher s/p rotator cuff surgery. They have been progressing very well through the acute, intermediate, and advanced stages of rehab and are now doing an interval throwing program to return to sport. However, the baseball season is fast approaching and the throwing program won't finish in time. The patient asks how you can speed up his progress and says he feels ready. What is the best way to help him? a. Watch him simulate some game pitches from the mound to see if he can do it b. Tell him that progressing the throwing program too quickly will drastically increase his re-injury risk c. Progress the program faster to the patient's symptom tolerance d. Consult his coach to evaluate how much playing time he will receive this season and adjust the program accordingly

Clinical Rationale: B is correct because gradual and criterion based Sx free progression prevents against athlete re-injury. A is incorrect because it is too aggressive and he could injure himself. B is incorrect because progression of the throwing program with the presence of symptoms will not properly prepare the pt. for sport. D is incorrect because his game play time is irrelevant to his return to sport, unless it is zero.

2) You are working with a young athlete to improve their shoulder / scapular stability and you have them do some slide board pikes. You notice excessive R scapular winging and you cue him to be aware of his form. Yet, after 3 more reps his scapula is winging again. What is the next best course of action? a. Decrease his exercise from 3 sets to 2 sets of 10 reps. b. Continue to cue him every time his scapula wings c. Regress his exercise to static planks where the winging ceases d. Continue with the exercise to strengthen his scapular muscles

Clinical Rationale: C is correct because he needs to be strengthening his scapular muscles with the proper form to avoid bad body mechanic habits. A is incorrect because he is already winging every 3 reps and decr. his sets won't help. B & D are incorrect because they are inefficient in time and ineffective in pt. rehab strategy.

A patient reports to you that he feels the most pain in the "cocking phase" of throwing when playing baseball. All of the following interventions would be appropriate to address this pain at the acute phase of rehab, except for: A) 90/90 doorway stretch of the anterior capsule B) Shoulder submaximal isometrics: IR/ER C) Rhythmic stabilization at the scapula D) Alternating isometrics of the UE at 90 degrees of scaption

Clinical Rationale: Individuals who experience pain with the cocking phase of throwing usually have anterior instability, therefore Answer A , a 90/90 doorway stretch of the anterior capsule, would not be an appropriate treatment intervention as this would continue to stretch out an already lax capsule. This patient would benefit from increased shoulder stabilization which is accomplished with answers B,C, and D.

Patient is a 23 y/o female reporting to PT with signs and symptoms consistent with R ITB syndrome. Which of the following exercises would be the most effective to strengthen the pts glutes? A. Side-lying clams with a resistance band B. Standing weighted hip abduction C. Lunges D. Hip hikes

Clinical Rationale: The correct answer if A. Clams have been shown to be the best exercise to target the glutes while minimizing TFL activation. This is important with this patient as activating the TFL as compensation for a weak glute medius can cause ITB tightness and increase the pain. Although B, C, & D also activate the glutes there is a higher incidence of TFL firing with these exercises as well that should be avoided with this patient.

A 45 y/o male patient is sent to you for PT eval and treat of "L heel pain". Upon your exam, you find he has pain along the distal 1/3 of his Achilles tendon. He also complains of pain when you measured his DF ROM of his L foot. Patient also complains of increased pain with resisted PF of the L foot. Patient reports that he feels limited in plantarflexion but that maximal PF is not the source of his pain.The patient notes that this pain has been going on for about 3 months. X-rays are negative. There is no noticeable swelling. Which of the following diagnoses more specifically describes this patient's symptoms? A) Achilles tendonitis B) Achilles tendinosis C) Posterior ankle impingement D) Os trigonum

Clinical Rationale: The correct answer is B) Achilles Tendinosis. Achilles tendonitis would not be an appropriate answer for this case as the patient has been experiencing these symptoms for the last 3 months. A tendonitis would only last a couple of weeks and have a more acute presentation. Two major symptoms of posterior ankle impingement pain are pain with maximal plantarflexion and swelling, which are not what the patient presents with. In addition, the location of the pain at the Achilles tendon also points to an Achilles pathology. Os trigonum is relatively rare in patients, only seen in about 5-10% of people. A common symptom is pain during plantarflexion . Someone with os trigonum would usually have areas of tenderness at the back and lateral side of the ankle. Swelling is also common with this pathology, as well as a hard lump near the Achilles tendon. As the patient demonstrates no swelling and palpation did not find a hard lump it is more likely that the patient has Achilles tendinosis. In addition the x-rays

Patient is a 36 year old male who presents to PT with pain in the back of his L heel with running for 1 week. At initial evaluation, patient demonstrates significant tightness of B gastroc/soleus complex, tenderness to palpation of posterior heel, noted swelling, but no thickening of the Achilles tendon was noted. When evaluating his running, you notice decreased DF and hip extension leading to significant vertical displacement. Patient joint mobility was determined to be 3/6 globally. You determine this patient has tendonitis of the Achilles tendon at the insertion. Based off this information, what would be an appropriate treatment plan for this patient: a. Reduce inflammation, stretch gastroc/soleus complex and hip flexors, eccentric strengthening of gastroc to neutral, glute strengthening b. Reduce inflammation, stretch gastroc/soleus complex and hip flexors, eccentric training of gastroc into DF, glute strengthening c. Gastroc/soleus and hip flexor stretching, posterior strengthening, eccentric strengthening of gastroc to neutral, glute strengthening d. Gastroc/soleus and hip flexor stretching, posterior strengthening, eccentric strengthening of gastroc into DF, glute strengthening

Clinical Reasoning: Correct answer - A. Seeing that symptoms are acute and appear to be a tendonitis and opposed to tendinosis, we want to begin by reducing the inflammation on the insertion of the Achilles. Based off the patients running kinematics we can assume due to the lack of DF and hip extension, a lot of load is being placed on the gastroc/soleus complex to accommodate for this leading to the vertical displacement which also stresses these muscles. Joint mobility is normal, therefore these restrictions in DF and hip extension are likely due to tightness of hip flexors and the gastroc/soleus complex. This narrows us down to answers a and b, the differentiating factor between these two is the amount of ankle DF we allow for with eccentrically strengthening the gastroc. Despite eccentrics being most commonly being prescribed for tendinosis of the tendon in order to remodel the tissue, these can also be used for this patient in order to accommodate for increased eccentric forces on the calf and Achilles with running with this vertical displacement. With insertional tendonitis/osis of the Achilles tendon, we do not want to go into DF as this places more stress on the insertion and could be disruptive to healing.

You are treating a 16 y/o female presenting to outpatient PT with patellofemoral pain syndrome (PFPS). Which of the following answers offers an appropriate treatment program for this condition? a. LE flexibility, patellofemoral joint mobilizations, balance activities b. LE flexibility, proprioception activities, gluteal activation exercises c. LE flexibility, eccentric calf strengthening, proprioception activities d. Gluteal activation exercises, proprioception activities, hip ROM exercises

Clinical rationale: Answer B, LE flexibility, proprioception activities, and gluteal activation exercises is the most correct response out of the listed choices. PFPS often develops as a result of LE tightness and proximal hip weakness, which leads to increased stress on the patellofemoral joint. Eccentric calf strengthening (mentioned in answer C) would be more consistent with a program for Achilles tendinopathy. Patellofemoral joint mobilizations, (mentioned in answer A) would not likely be a treatment to address why the knee pain was occurring. This would likely not be an appropriate treatment since it is usually not a limitation with this population. Hip ROM exercises (listed in answer D) would not likely be a main concern in this population, but rather limitations found in ROM are a result of muscle length imbalances, so treatment should be guided to address that.

Which of the following is an intrinsic risk factor for developing Achilles tendinopathy? a. Limited ankle dorsiflexion b. Foot pronation c. Abnormal subtalar joint ROM d. All of the above

Clinical rationale: Limited ankle dorsiflexion (a), foot pronation (b), and abnormal subtalar ROM (c) are all consistent with Achilles tendinopathy, which makes all of the above (d) correct. Limitations in subtalar ROM restrict the amount of calcaneal movement during the phases of gait, which changes what is happening up the kinetic chain and put more stress through the Achilles tendon. Increased foot pronation means that the gastroc muscle has to work harder to push off of an unstable foot during propulsion. Limited ankle dorsiflexion will lead to an earlier heel rise, which means that the gastroc complex has to work for longer and harder when trying to push off of that limb, which will put additional stress on the Achilles tendon.

You are performing a qualitative running analysis on a 35 y/o female that presented to your outpatient sports clinic. During this evaluation, you observe excessive vertical displacement, evidenced by "bouncing" that you see. Which of the following is most likely the reason behind this? a. Midfoot hypermobility b. Posterolateral hip weakness c. Restrictions in the sagittal plane d. Poor motor control

Clinical rationale: Midfoot hypermobility (a) is more consistent with the over-compliance running type. In this running form, there is increased pronation, as well as a valgus collapse up the kinetic chain. Posterolateral hip weakness (b) is also seen with over-compliance, as well as the crossover running type. In both of these running forms, there is proximal weakness that allows for increased femoral internal rotation or femoral adduction with running. Poor motor control (d) is also consistent with the over-compliance running form. In this type, there is a decreased ability of the limb to control the deceleration movements in the beginning of the gait cycle, which leads to a valgus collapse along the kinetic chain. This leaves restrictions in the sagittal plane (c) as the correct answer. Excessive vertical displacement is often a result of hip flexor or gastroc/soleus tightness, as well as limitations in talocrural mobility.

2. You are trying to decide which strengthening exercises to prescribe your patient. You decide to use findings from a recent study to make your decision and determine the best choice would be: Exercise A which has the most activation of the targeted muscle. Exercise B which has 4th greatest activation of the targeted muscle and the least activation of overactive muscles. Both exercises will have the same effect. Research regarding UE strengthening is unreliable and should not be considered when making clinical decisions

Clinical rationale: The best exercise to strengthen a particular muscle is not solely based on which exercise has the greatest activation of that muscle. Instead, it is better to choose an exercise that has the most activation of that muscle as well as the least activation of muscles that are already overcompensating for the strength deficit. Therefore, answer B is a better choice than answer A. Answer C is incorrect because exercise B would have a better effect than exercise A in this example. Answer D is incorrect because you should always use an evidence based approach when making clinical decisions.

1. Which of the following is true regarding the interval throwing program? Begin throwing on flat ground and progress to throwing from a mound. Begin by throwing on an arc and progress to throwing in a line. Begin by throwing fast balls and progress to curve balls. Both A&B are correct

Clinical rationale: The interval throwing program begins with throwing on a flat, stable surface before progress to a pitcher's mound, which is a more challenging surface. The program also begins with throwing on an arc and then progresses to throwing in a line. Answer C is incorrect because players do not throw fast balls until phase two and there is no progression to curve balls included in the program. Therefore, the best choice is answer D.

Which of the following is an intrinsic factor that may be contributing to running related injury? a. Training with too little rest b. Tight hip flexors c. Running on trails d. Shoes with worn out midsoles

Correct Answer: B. All other options mentioned here are extrinsic or environmental factors that can cause running related injuries. Tight hip flexors are related to poor flexibility and can contribute to limited peak hip extension during stance phase leading to faulty mechanics elsewhere.

Which of the following is NOT true regarding excessive vertical displacements with running? a. Can be caused due to calf tightness b. Secondary to limited subtalar mobility c. Places stress on patellofemoral joint d. You will see excessive knee flexion

Correct Answer: B. B is false because it is often secondary to limited talocrurual mobility. With limited dorsiflexion, you often see an early heel rise contributing to the bounce and increase in vertical displacement. A is true as that is commonly also limiting dorsiflexion. C is true with the increased impact peak and loading leading to stress at the knee.

Which of the following is not required for clearance of strength testing? a. <1/10 knee pain b. Active flexion with patella glide c. Full ROM d. Minimal joint effusion

Correct Answer: B. B is false because you want there to be active hyperextension with a superior patellar glide. This indicated the patient has enough control and can't actively reach their ROM, which is important to ensure prior to focus on strengthening. A and D are true because you want to ensure that pain or swelling are not limiting their ability to perform. C is true because you want full ROM prior to strengthening.

You have a patient that comes in with anterior knee pain and presents with increased foot pronation, decreased knee flexion angles, and coxa valga. This person is at risk for what? a. ITB syndrome b. Achilles tendinopathy c. Patellofemoral Pain syndrome d. Plantar Fasciopathy

Correct Answer: C. Excessive foot pronation leads to tibial and femoral IR, increasing contact on lateral facets of the patella and leading to knee pain. Femoral inclination changes the pull of the glut med leading to reduced ability to generate force. Though the patient may present with these other conditions, the following risk factors as well as the anterior knee pain, will lead to a hypothesis of patellofemoral pain syndrome.

1) Patient is a 24 year old who has started training for a half marathon who has been running recreationally for 6 years. Patient presents with pain vertically up R tibia with running and jogging and has history of L tibial stress fracture. Upon evaluation patient presents with increased weight bearing on medial side of foot, increased foot pronation and, significant navicular drop, and tenderness to palpation going up medial shin. No edema or numbness and tingling reported. Radiograph and MRI were negative. Based off this information, what diagnosis does this patient most likely present with? a. R tibial stress fracture b. R gastroc strain c. R medial tibial stress syndrome d. Compartment syndrome

Correct answer - C. Based off the information provided, the foot mechanics and increased training could be indicative of potentially either a stress fracture or MTSS. The differentiating factors here are the location of the pain; vertical pain up medial shin as opposed to pain horizontally across the tibia, and the negative MRI rules out a stress fracture. No edema or numbness and tingling anywhere also rules out compartment syndrome which would present in the anterior compartment of the leg.

When performing a running evaluation on a patient who presents with gradual onset R knee pain with running, you notice that during stance phase the R leg has greater genu valgum than the L, popliteal fossa's facing laterally, significant hip drop, B crossover of the swing legs, and excessive forefoot pronation R> L. When assessing the patient's feet, you establish the patient's STJ is in normal alignment, DF is WNL, and hypermobility of the midfoot. Patient has R knee pain that has stopped him from running, has 5/10 pain with ascending and descending stairs and squatting activities, and 3/10 pain at rest. The most appropriate exercise prescription for this patient at this time would be: a. SL hip abduction, clamshells, piriformis stretch, calf raises b. Single leg squats, lateral band walks, hip adductor stretch, post tib strengthening c. SL hip abduction, step ups, hip adductor stretch, post tib strengthening d. SL hip abduction, clamshells, hip adductor stretch, post tib strengthening

Correct answer - d. The excessive pronation, genu valgum, and popliteal fossa's facing laterally suggest the hip is in IR and the ER's are doing a poor job of keeping the femur in a neutral position. Therefore, we would want to strengthen the hip ER's in order to reduce the femoral IR, the clamshells are the exercise of choice in this case as they directly target the hip ER's. The hip drop and crossover are indicative of weak hip abductors. Despite SL hip abduction not being as functional in promoting dynamic control of the hip abductors like performing SL squats or step ups, these target the hip abductors more for strengthening purposes. For this reason, as well as the patient's subjective complaints of pain with squatting and stairs, SL hip abd would be the most appropriate for the patient at this time. The crossover sign may be indicative of tight hip adductors pulling the femur into adduction, either this or due to imbalance between hip abductors and adductors this is placing more repetitive stress on the adductors causing them to inherently get tighter. Therefore, stretching these would be more appropriate than stretching the already weakened hip ER's with the piriformis stretch. Posterior tib strengthening would be recommended over calf raises seeing that the patient has a hypermobile midfoot allowing for excessive pronation. By strengthening the posterior tib this would help control the excessive pronation at the midfoot.

2. You noticed your patient has significantly weak gluteal muscles. You are trying to create an exercise program for him, but you aren't sure which ones will mainly target the gluteal muscles over TFL. You ask Dr. Nolan what he thinks and he responds: A) Sidelying hip abduction B) Lunges C) Clamshells D) Squat

Correct answer is C. According to Selkowitz et al., clamshells and squats are the only two exercises on this list in which both gluteal muscles demonstrated significant higher normalized electromyographic signal amplitude than TFL. However, clamshells had a gluteal-to-TFL activation index of 115, versus a 28 gluteal-to-TFL activation index for squats. Gluteal-to-TFL activation index for sidelying hip abduction= 38; gluteal-to-TFL activation index for lunges= 18.

. Which of the following is NOT a risk factor for patellofemoral syndrome? A) Femoral structurel B) Decreased knee flexion angles C) Excessive foot pronation D) Increased knee flexion angles

Correct answer is D. A, B, and C are true risk factors for patellofemoral syndrome. Femoral structure can be a risk factor- femoral inclination (coxa valga) can reduce gluteus med moment arm. Femoral ante version can lead to IR of femur and tibia, causing increased stress on PF joint. Decreased knee flexion angles leads to decreased contact area of patella. Finally, excessive foot pronation will also lead to tibial IR, which then leads to femoral IR and increases contact pressure on lateral facets of patella.

Lisa comes into your outpatient clinic with knee pain. She says she has been training for a marathon but hasn't increased her mileage in a while due to her pain. You decide you want to analyze her running mechanics. You observed cross-over. Which of these is true? A) Posterolateral hip weakness B) Toe-in/foot inversion C) Foot crosses over midline D) Lateral tibia is being stressed E) A&C are true F) B & D are true

Correct answer is E (both A & C are true). With crossover, we will see four things: foot crossing over midline, toe-out/foot eversion, lateral pelvic tilt, and pronation. These are occurring due to posterolateral hip weakness and adductor tightness. Due to these running mechanics, the medial tibia, tibialis posterior, and the femoral neck are being stressed.

3. What is the functional testing algorithm? a. Sport-specific progressionsàFunctional testingàCKC Testingà OKC Testingà Proprioceptive testingà Basic measurement b. Basic measurementà Proprioceptive testing à OKC Testingà CKC TestingàFunctional testingàSport-specific progressions c. Basic measurementà Proprioceptive testing à CKC Testingà OKC TestingàFunctional testingàSport-specific progressions d. Proprioceptive testingà Functional testingà OKC Testingà CKC Testingà Basic measurementà Sport-specific progressions

Correct answer: B Rationale: Establishing testing is important prior to sport specific progressions. According to notes and lecture, OKC testing should be established prior to closed kinetic chain. Open chain gives a more accurate depiction of muscular control and strength, wheras closed chain is more protective allowing for better stability.

If you wanted to work on improving neuromuscular control in a patient to decrease the risk of ACL injury, which of the following would be least important to focus on? a. Joint position with landing a jump b. Knee valgus with squats c. Strength of the quads d. Co-contraction of quads and hamstrings

Correct answer: C. A is true because you want to work on proprioception and joint position to improve the ability to tell where you are in space and reduce the risk of faulty mechanics with landing. Improving knee valgus with squats helps with better control and improved joint position to prevent falling into that injury prone position. D is true because contracting the hamstrings and quads help to create a stable knee with emphasis on control. Though strength of the quads is important, it would be the least important to focus on because it would not help with neuromuscular control. Therefore, C is false.

Which of the following about meniscal root tears is FALSE? a. Essential role is to dissipate axial load to hoop stress b. Meniscal root tears and avulsions are like a total meniscectomy c. Strictly NWB for 6 weeks d. Radial force from axial load results in intrusion

Correct answer: D. Radial force from the axial load results in extrusion. As choice A states, the axial load is dissipated as hoop stress. With a root tear, there is no peripheral hold of the meniscus and therefore Extrusion can occur. Doherty, D. Lowe, W. Meniscal root tears: identification and repair. Am J Orthop. 2016 March; 45(3):183-187.

1. Which of the following is NOT true during the acute phase of rehab for an overhead athlete? a. Goal is to restore normal ROM b. Closed chain kinetic activity should be avoided c. Evidence for modalities is limited d. Focus manual therapy on IR/ER

Correct answer: b. Rationale: During the early stage of rehab, the most important goal is to restore normal ROM, as that is difficult to achieve after the healing process has progressed. Closed kinetic chain activity should not be avoided because proprioceptive input will help with pain and inflammation as well as initiating neuromuscular control. Evidence with modalities is limited in the research, and varies based on patient perceptions of benefit. Manual therapy focusing on IR/ER is important to ensure adequate ROM in all directions, since many overhead athletes require excessive rotation.

3. Which component would be most important to include in the advanced stage rehab of a skier who had a knee arthroscopy? A. Maintaining a squat on a compliant surface with eyes closed B. Single leg balance with external perturbations C. Cutting drills D. Weight shifts in standing to simulate turns

Correct: This is the most sport specific drill that will encourage stability and strength throughout the knee joint. Maintaining a squat position is needed for skiing, as well as adapting to uneven surfaces. Closing the eyes can simulate decreased visibility in snowy conditions. It is important to incorporate sport specific drills to be sure the athlete is ready to handle the specific challenges of their situation before returning to the actual environment.

2. Josephine, an 85 y/o sedentary female, is coming in to see you for rotator cuff impingement. You tried to have her do side-lying ER and that was too easy for her. Which is the best recommended way to progress this? A) Standing ER at 90 degrees B) Prone ER C) ER at 0 degrees with towel roll D) ER in scapular plane

D is CORRECT. According to Reinold et al. JOSPT 2004, ER at 90 degrees of abduction is a functional position for overhead atheletes/workers. Josephine, is an 85 y/o sedentary female who would benefit more from a neutral position, instead of 90 degrees of abduction. ER at 90 degrees of abduction increases strain on the capsule and should be avoiding in early rehab. ER at 0 degrees with towel roll is the first step of progression, prior to side-lying ER (which is what she attempted and was too easy for her). After side-lying ER, the next progression step is ER in scapular plane. Scapular plane is the most functional position for our daily activities.

Gary presents to your clinic post humeral fracture 9 weeks ago. He demonstrates forward flexed posture with tightness in pec major/minor and weakness in posterior musculature. You want to target his lower serrates anterior specifically. What exercise is the least likely to target just the lower serrates anterior? A) Shoulder flexion B) Push up plus C) Scaption D) Military press

D is CORRECT. The MVIC for military press is 60%; MVIC scaption= 84%; MVIC shoulder flexion= 73%; MVIC push up plus= 73%. MVIC= Maximal voluntary isometric contraction

Mary is a 40 year old female, coming to your outpatient PT clinic with bilateral knee pain. Mary states that she recently started training for a half marathon and her knee pain is progressively getting worse. After the subjective portion of your evaluation you are suspecting that Mary may have patellofemoral pain syndrome. Which of the following would you expect to see during your objective tests and measures? a. Hip abductor weakness b. Hip external rotator weakness c. Excessive foot pronation d. All of the above

D is correct because all of the above are risk factors for patellofemoral pain syndrome. Both hip abductor and hip external rotator weakness allow the patient's femur to adduct and internally rotate putting increased stress at the knee, as well as proximal and distal joints. Excessive foot pronation leads to femoral and tibial IR, which in turn increases contact pressure on the lateral facets of the patella.

After performing more objective measures you find that John has increased bilateral rounded shoulders, forward head posture, general 4+/5 right periscapular strength, 5/5 left periscapular strength, and bilateral UE ROM is WFL. What is an appropriate intervention for John? a. Perform a manual bilateral pectoralis minor stretch. b. Right shoulder IR and ER with TheraBand. c. Supine serratus punch d. All the above are appropriate interventions

D is correct. Bilateral shoulder protraction/bilateral rounded shoulders can be seen secondary to pectoralis minor tightness due to the orientation of the muscle. Additionally, John performs repetitive overhead motions and is a student, both of which can affect posture. Due to decreased right periscapular strength B is correct. Increasing pericapsular strength/retraction will improve his posture/alignment. Finally, protracted shoulders can be due to weak serratus anterior muscles, therefore strengthening serratus anterior with serratus punches will improve John's impairments. As a result of the many possible impairments, all of the above options are correct.

A high school baseball pitcher presents to your clinic with complaints of minor pain while pitching along with decreased pitch speed and decreased accuracy. When you ask him how long this has been going on for he replies "over the past couple weeks of games my performance has been declining". Upon further subjective questioning patient reports that the most provocative position in with the arm in full ER of the cocking phase. What would you expect to find with your objective assessment? a. Decreased ER and increased IR b. Anterior capsule tightness and posterior capsule laxity c. Increased humeral anteversion d. Increased ER and decreased IR

D is the correct answer because baseball pitchers on the dominant pitching hand tend to have increased ER but decreased IR keeping the ROM within the arch of 180 deg. This allows for the pitcher to have a greater amount of torque to increase velocity of the pitch. A and B are related because if there was anterior capsule tightness there would be less ER and the same goes for increased posterior capsule laxity, the greater the posterior laxity is the greater IR will be. This would be inefficient for a pitcher based on the pitching mechanics. Answer C is incorrect because if the humerus was anterverted there would be decreased ER and increased IR. Answers A-C are all interrelated and not mechanics that are typically associated with baseball pitchers.

You are working with a high school female athlete who presents with knee pain, glute weakness, decr. hip extension, and knee valgus. Her pain has been keeping her from playing basketball, but she is eager to return to sport. After running her through basic strengthening exercises, what is the next best step to make sure she is also learning proper body mechanics? Have her return to playing basketball to functionally work on her deficient body mechanics Have her watch recordings of herself playing basketball to learn where she is deficient in her body mechanics Have her play another sport to challenge her to use different body mechanics Give her simplified movements to do in front of a mirror to relearn proper mechanics

D is the correct answer because by simplifying her activities, she can focus on specific body mechanics that are weak/deficient and re-learn how to do them properly in isolation. A and B are incorrect because full gameplay requires too many movements to isolate the one's that are weak/deficient and improve on them. C is incorrect because if returning to basketball is her ultimate goal, improving body mechanics in other sports will not address the mechanics that she will be mainly using in the future

Your patient is a post-OP meniscus repair in the Early phase of rehab. They tell you that they've been having a hard time getting in and out of cars with the leg brace locked, and ask you if it is OK to unlock the brace to get out. After all, they tell you, the doctor said it's ok to be TDWB-WBAT. How should you educate them on why this is a bad idea. The pt.'s leg could get caught on something and bending the knee too soon could delay healing or disturb the surgery site The pt.'s leg muscles are too weak and they need the brace to support their weight In order to heal properly the knee needs to remain immobile for as long as tolerable The meniscus moves with knee bending and putting weight through it at the same time could disturb the surgery site

D is the correct answer because imaging shows that the meniscus moves with knee flexion to accommodate the femoral condyles. Putting weight through the meniscus in this position can cause excursion which can impact healing of the surgery site. A is incorrect because bending the knee alone is not bad for a post-OP meniscal repair. B is incorrect because the knee brace isn't supporting the pt.'s weight when they standing. C is incorrect because a pt. should not keep their knee immobile for as long as tolerable after a meniscal repair.

3) During the initial eval, your pt. tells you she has been having a hard time lifting her R arm over her head and she presents with impingement signs when you test her AROM. What is the best test to perform next that will give you the most relevant information regarding her Sx? CKCUEST Seated Shot Put Test Lateral scapular slide test Scapular Repositioning Test

D is the correct answer because it would immediately give you positive feedback and diminish the Sx, pointing to scapular positioning abnormalities or strength/mechanical issues. A & B are incorrect because they are more functional tests and would only increase the Sx / compensations, but not tell you what the issue was. C is incorrect because while it would tell you that there was a dysfunctional scapula, it wouldn't also shed light on where to go with treatment.

Thomas is a 45 year old male who presents to your outpatient PT clinic with pain with palpation along distal 1/3 of right achilles tendon, pain with passive dorsiflexion, pain with resisted plantar flexion and reports of pain during basketball and softball leagues with his co-workers. You determine that Thomas has right achilles tendinopathy due to his reports of increased pain over the last year and repetitive microtrauma to the tendon. Which of the following are appropriate interventions for Thomas? a. Single leg balancing on stable and unstable surfaces, eventually progressing to internal and external perturbations b. Eccentric gastroc/soleus exercises c. Tibialis posterior strengthening d. All of the above

D is the correct answer, all of the above interventions are appropriate. Single leg balancing will incorporate foot intrinsics, allow for co-contractions and improve neuromuscular control of Thomas' lower extremities. Eccentric gastroc/soleus exercises have been shown to correlate with improved outcomes by increasing tendon load and strengthening. Tibialis posterior strengthening is important to further improve Thomas' plantarflexion strength and controlling his arch during weight bearing activities.

Pt is 9 weeks s/p L central meniscus repair. Which of the following would not be an appropriate intervention to perform with the pt in the clinic at this time? A. Gait training with a focus on heel strike and terminal knee extension B. Prone weighted HS curls C. DL squats on BOSU D. An interval jogging program

D is the correct answer, as a jogging program is usually not started until the late post operative phase unless the pt had full ROM, no swelling, normal and pain free gait, and 80% quad symmetry. Gait training would still be important at this time so that the pt can progress towards jogging. HS strengthening can also be initiated in this intermediate phase along with BOSU squats which are CKC strengthening on an unstable surface.

Becca comes in following an ACL reconstruction. She is asking you if she can start jogging again. What is the main component you are looking for in order to clear her? A) Passive hyperextension with superior patellar glide B) Trace or less joint effusion C) 2/10 knee pain D) 80% limb symmetry

D is the correct answer. When calculating MVIC, you calculate it in 60 degrees of knee flexion (quad neutral) and then compare it to the uninvolved side to calculate limb symmetry index. If the patient has at least 80% limb symmetry they are cleared for jogging and hop testing. Clearance for testing is ruled by 4 items: full, pain-free ROM, active hyperextension, 0-1/10 pain, trace or less joint effusion.

. Which of the following is the most important factor to achieve first in the rehabilitation of an overhead athlete? A. Muscular strength/endurance B. Proprioception C. Dynamic stability D. Restore ROM

D is the most appropriate answer because ROM must be restored before the shoulder has the chance to scar down, as the ROM is very important for proper throwing mechanics. ROM is the most difficult to attain overtime and therefore should be the first priority. A, B, and C are all other important factors that are necessary in order to return to sport and to prevent re-injury but can be addressed as secondary concern to ROM.

A right-handed professional baseball pitcher presents to the PT for evaluation of his R shoulder. Which of the following ROM measurements would not be expected for an overhead athlete? A. R shoulder ER at 90 degrees of ABD: 60 degrees B. R shoulder IR at 90 degrees of ABD: 120 degrees C. R shoulder ER at 90 degrees of ABD: 110 degrees D. A and B

D. A and B Clinical Rationale: As this patient is an overhead athlete, it would be expected for his dominant arm to have more ER ROM than IR ROM. As this is specially a baseball pitcher, he requires a great amount of ER when throwing. In order to maintain this arc of motion, it would be expected that his IR ROM was less than the normal range. Therefore a more "normal" finding would be A and B reversed: ER= 120 degrees and IR=60 degrees. Answer C would be an expected ER ROM measurement of a baseball pitcher. Therefore, the correct answer for this question is D.

3. 22 yo F collegiate soccer player is 6 months s/p R ACLR. Which of the following strengthening interventions is most appropriate for sport specific training? a. Uneven surface training b. Eccentric hamstring loading c. Concentric quadriceps strengthening d. All of the above

D. All of these interventions are appropriate for soccer specific sports training/strengthening.

1. 29 yo F patient presents to clinic s/p L knee medial meniscal root repair 2 weeks ago. Patient is currently NWB using crutches and does not wear a brace. Which of the following is the MOST appropriate intervention? a. Aggressive hamstring stretching b. Passive knee ROM 0 - 120o c. Initiate WBAT d. E-stim assisted quad sets

D. Patient presents in the protective phase (0-6 weeks) after surgery. Aggressive hamstring stretching is not appropriate at this time due to the anatomical attachments to the medial meniscus. Knee ROM past 90o is not appropriate because the meniscus moves during flexion/extension and could disrupt the repair. WB is not appropriate until the intermediate phase of rehab due to hoop stress.

2. 20 yo college baseball pitcher presents to clinic c/o L shoulder pain. Patient is L handed and reports shoulder pain started while pitching in game 5 days ago and has not improved. Currently unable to throw or reach overhead without reproducing symptoms. Reports he has been playing baseball since he was a kid. Evaluation reveals 118o of ER and 60o of IR at 90o of abduction. Weakness of middle and lower trapezius muscles, posterior rotator cuff muscles, and serratus anterior. Patient is able to replicate UE positions with eyes open but consistently 10 - 15o off with eyes closed. Which of the following interventions would be the MOST appropriate for the next treatment session? a. Manual stretch of external rotators b. Grade III-IV posterior and inferior GH glides c. With patient's eyes closed, position patient's R UE in 45o of ABD and 30o of ER and ask patient to replicate position with L UE. Provide feedback as needed. d. With patient's eyes closed, position patient's L UE in 45o of ABD and 30o of ER. Return arm to neutral and ask patient to replicate position with eyes closed. Provide feedback as needed.

D. Patient presents with decreased proprioception which puts him at risk for injury due to the inability to recognize injury provoking positions. Answer D is a good starting point to help retrain his spatial body awareness. Answer A in incorrect because the patient presents with normal ROM measurements for a competitive baseball pitcher. Trying to increase IR by stretching the external rotators will increase his total arc of motion beyond 180o and cause further instability/pathologies. Answer B is an appropriate treatment intervention but it is not the best choice. Answer C is slightly more difficult than answer D and would be a good progression for future visits as the patient improves.

2. Which of the following would not apply when making a training program for a high school cross country athlete? a. Loads between 60-80% of 1 RM b. Short rest periods c. 6-12 repetitions d. 12-20 repetitions

Endurance training is done for athletes that require energy/stamina for a long duration of time. Endurance training is defined as loads between 60-80% of 1 RM, short rest breaks, and high repetitions. Based on this, the answer is C. Answer C would apply to strength training. When creating training programs, for athletes especially, it is important to take into account the requirements of their sport. Endurance athletes need to sustain their energy over a long period of time, so this type of training works best for their sport.

2. After several weeks of physical therapy for her shoulder, a softball player is beginning to return to her sport. You initiate an interval throwing program. The focus of this program should be: a. Increasing maximal velocity of the throw at short distances b. Throwing maximal distances pain free c. Throwing without an arc trajectory d. Maintaining proper throwing mechanics as throw distance progresses

In an interval throwing program after shoulder rehab, the goal is to gradually build the shoulder back up to appropriate strength and retrain proper motion to throw successfully and prevent further injury. Velocity and distance are not important until proper mechanics are achieved and pain is decreased so a and b are incorrect. The correct answer is d. In an interval throwing program after shoulder rehab, athletes should perform a progression of flat ground throwing, focusing on proper mechanics at each stage to prevent further injury, and throwing pain free on an arc before progressing the distance.

Which of the following exercises is not appropriate for a patient with noted posterior instability of the R glenohumeral (GH) joint? a. High plank hold on slide board with L arm circles b. R single-arm plank on BOSU with throw/catch of L arm c. R single-arm plank reaching L arm under/across body to R side d. High plank with alternating hand lifts

In correct answer is C. A, B, and D all describe closed-chain stabilization exercises for the shoulder girdle, which would be appropriate for this patient. Option C describes an exercise that will put increased strain on the posterior capsule of the GH joint, which is a vulnerable position for a patient with posterior instability of the GH joint.

You are working with a 40-year-old female s/p meniscal root repair. Which of the following would you not want to work on during the first 6 weeks after surgery? a. Quad strengthening b. Gait training with one crutch on affected side c. Grade 3-4 patella mobilizations to restore knee mobility d. Passive and active ROM to restore full knee extension

It important to maintain all precautions following a meniscal root repair. The biggest precautions are strict NWB status, no isolated hamstring contraction or hamstring stretching for the first 6 weeks, and restricted knee ROM 0-90 degrees for the first 4 weeks. Gait training with one crutch would be breaking the NWB precaution. We can work on gait training with 2 crutches during the first 6 weeks to teach the patient how to properly use crutches and how to best conserve energy. All the other answers are things that we would want to do to put the patient in the best position for success after the repair.

During your initial evaluation of a recreational runner you determine the patient has a (+) Thomas test, gastroc and soleus tightness, and talocrural mobility is 1/6 (all findings are bilateral). What would you expect to see during a running analysis? a. Crossover running b. Excessive supination during loading response c. Excessive vertical displacement d. Nothing, as all these findings are normal of a runner.

Joint and muscular restrictions in the sagittal plane lead to excessive vertical displacement with running. All of those restrictions mentioned control sagittal plane movements leading to a "bouncy" running type. Therefore, the correct answer is C. Crossover running is caused by posterolateral weakness. Answer B is incorrect as we would expect to see excessive pronation throughout the entire gait cycle in order to compensate for a lack of dorsiflexion. Answer D is also incorrect as these running mechanics will most likely lead to injury.

While observing Mike walk you note an early heel rise (bilaterally), excessive knee flexion, and a "bouncing" gait. What could be contributing to Mike's gait abnormalities? a. Hip extensor tightness b. Gastric/soleus tightness c. Hypermobile talocrural mobility d. Mike's gait is "normal"

Option A is incorrect because hip extensor tightness would restrict active hip flexion, which is not an issue for Mike. It is possible that the hip flexors may be tight which restricts hip extension. If Mike does not have adequate hip extension while he is advancing over his stance limb, his body would compensate with an early heel rise and excessive knee flexion. Option B is correct because if the gastric/soleus complex is tight this would not allow his ankle to reach optimal dorsiflexion, creating a compensation of excessive knee flexion. Option C is incorrect because a hypermobile talocrural joint would not contribute to a "bouncing" gait, however a hypomobile talocrural joint would. If the joint was hypomobile it would decrease Mike's ability to dorsiflex his ankle during the stance phase of gait. Option D is incorrect because there are impairments/abnormalities with Mike's gait that should be addressed.

2. Which exercise/intervention is not appropriate for a patient in the early stages of rehab who had a meniscal and acl repair? · a. supine quad sets · b. patellar mobility · c. short arc quads · d. restore extension ROM

Rationale: Correct answer C. If the ACL and meniscus are injured and repaired at the same time rehab may vary a bit in the beginning stages due to the ACL repair. Short arc quads put stress on the knee joint in a way that may harm the repair.

1. The medial meniscus has attachments to which structure(s)? · a. MCL · b. popliteus · c. semimembranosus · d. a & c

Rationale: Correct answer D. It is important to know what structures attach to the medial meniscus because those structures could have also gotten injured. Being aware of that can help to guide the examination and treatment.

3. What criteria must be met before a jogging program can be initiated at PT in patients with meniscal repairs? · a. full ROM · b. trace or less knee effusion · c. quad symmetry of 80% · d. normal pain free gait · e. all of the above

Rationale: Correct answer E. This is important criteria to note so that you know when it is safe to introduce jogging to the PT program. The progression starts with small time increments of jogging to introduce the forces of jogging to the joint.

3. During the intermediate phase of rehabilitation for the overhead athlete with a shoulder injury, lower body strength and balance can be introduced to assist with which of the following? a. producing, transferring, and dissipating forces b. strengthening lower body and increasing balance are not appropriate for this patient because they are being treated for a shoulder injury c. motor control and prevention of re-injury d. a and c

Rationale: Correct answer is D. Strengthening lower extremities and improving balance can help improve motor control and prepare the body to handle unexpected forces. This also helps the athlete to make efficient overhead movements to help produce, transfer, and dissipate forces adequately enough to prevent re-injury.

2. Before having a pt perform an UE functional test in closed kinetic chain what are the most important things to screen for? a. strength and A/PROM of the shoulder b. scapular control c. No particular screens are advised prior to testing d. a and b

Rationale: D is the correct answer. The CKC tests can create a heavy load through the shoulder, wrist, and elbow, as well as create large amounts of posterior shoulder stress. If you are unaware of your pt's areas of weakness, instability, or pain you could end up causing increased damage to the patient.

When introducing new running techniques to patients it is important to remember which of the following? · a. make small, gradual changes · b. monitor for new injuries · c. land with foot near the body's center of mass · d. all of the above

Rationale: The correct answer if D, all of the above. It is important to make small, gradual changes so that the body has time to adapt to these new mechanics. At the same time it is also important to monitor for new injuries that may arise due to these new mechanics and tailor treatment accordingly. Having the patient keeping their feet near the bodies center of mass while running will help to put them in a better position to absorb forces will running and keep them in a optimal position to not stress other structures which can also cause injury.

2. What types of diagnoses are important to tease out when trying to determine if you patient has medial tibial stress syndrome? · a. Compartment syndrome · b. Stress fracture · c. a & b · d. none of the above, nothing presents like medial tibial stress syndrome

Rationale: The correct answer is C, a and b. It is important to be able exclude compartment syndrome and stress fractures from medial tibial stress syndrome because they are all treated differently. If a patient has compartment syndrome they should report different symptoms such as numbness or tingling in their foot or swelling due to this largely being a neurovascular issue. Patients who have stress fracture will experience pain in a horizontal pattern and patients with medial tibial stress syndrome will have diffuse pain along >5cm of the posteromedial tibia in more a vertical pattern.

1. Each of these are risk factors of Patellofemoral Pain syndrome EXCEPT? · a. age >34 y/o · b. High Q angle · c. Hip abductor weakness · d. Hip adductor weakness

Rationale: The correct answer is d. hip adductor weakness. Being female and, hip ER weakness, and increased knee flexion at midstance are also risk factors. It is important to be aware of these risk factors to know what population might commonly present to you with patellofemoral pain syndrome. Risk factors such as a high Q angle and hip abductor weakness can cause abnormal force at the patellofemoral joint which can cause pain due to not absorbing force properly.

If a patient is unable to successfully perform a push-up plus in high plank for a challenge to the serratus anterior (SA), which of the following modifications is most indicated? a. The patient should perform a push-up plus in a modified push-up position with their knees on the ground. b. The patient should perform a push-up plus against a wall. c. The patient should perform as many push-ups plus in high plank as they can or the SA will not get stronger. d. The patient should perform a dynamic hug without resistance.

The correct answer is A. Option B may result in significant activation of the upper trapezius muscle, which is not a goal of this exercise. Option C is incorrect; a patient performing an exercise with compensation does not serve the patient well. It is better to modify the exercise to successfully challenge the targeted muscle(s). Option D would target the SA, but may be an exercise that is indicated if the patient were unable to perform a modified push-up plus. Option A describes a modification to this exercise that will target SA without causing excessive activation of the upper trapezius.

A 30 year old female comes to your clinic complaining of pain that goes up and down her lower leg. She reports she began running within the last few months in hopes to lose weight. During your running analysis you notice that she pronates during the entire gait cycle. Which of the following is the most likely diagnosis? a. Medial tibial stress syndrome b. Stress fracture c. Compartment syndrome d. Patellofemoral pain syndrome

The correct answer is A. Risk factors for MTSS include female gender, low activity history, BMI>20.2, and tibial acceleration. Based on these risk factors and this patient's history and examination, the likely diagnosis for this patient is MTSS. A stress fracture would be more localized, horizontal pain. This patient is complaining of more diffuse, vertical pain which leads us to believe MTSS over a stress fracture. Since there is no history of trauma to the lower leg and the patient doesn't report any neurological pain (numbness, tingling), it is most likely not compartment syndrome. Pain from PFPS will be in the knees and tends to be more common in older women.

1. You are working with two patients, one s/p contact ACL injury and one s/p non-contact ACL injury. Which patient would you expect to take more time to return to sport? a. Contact injury because the patient might get hit again and suffer another injury. b. Non-contact injury because there is most likely an underlying weakness or instability that needs to be addressed. c. Non-contact injury because the surgical repair procedure is more invasive. d. Both should return to sport in the same amount of time.

The correct answer is B. Athletes who suffer non-contact ACL injuries tend to have an underlying instability or weakness that needs to be corrected in order to safely return to sport. It is important that these deficiencies are addressed by a physical therapist to prevent re-injury. Re-injury rates are higher than we would like so it is vital to make sure that patients are in the best condition for preventing re-injury before allowing them to return to sport. Answer A is a concern, but if contact is part of their sport, it is something we cannot prevent. We can still strengthen the patient to help prevent re-injury, but it is not a reason to hold an athlete from returning to sport. Answer C is false. The surgical procedure is not different based on the type of ACL injury. Answer D is false as well. Patients s/p non-contact ACL injury tend to have a more extensive rehab process.

2. A PT prescribes a exercise program for a baseball player which incorporates catching, throwing, and agility workouts in a baseball field. The patient is currently in the accumulation phase of this exercise program. What principle of periodization does this exercise most closely align with? A) Linear B) Non-linear C) Undulating D) Block

The correct answer is D) Block. The block principle focuses on highly concentrated, specialized workloads that does not focus on components of training that the athlete does not need. The block principle can be separated into 3 distinct phases: accumulation, transmutation, and realization. Answer A is incorrect because the linear principle focuses on changing volume and load predictably and is used when the individual wants to work towards one major competition, which would not apply to a baseball player. Answer B and C are both names for the same principle. The non-linear/undulating principle focuses on volume and load are altered frequently and there are frequent changes in stimuli as well as greater recovery time.

You are assisting a pitcher with return to sport. The patient now demonstrates significant strength with side-lying external rotation (ER) while using a towel roll. Which of the following is the most appropriate progression to address this patient's demands of sport? a. Continue to increase weight used in this exercise in the same position until the patient is able to perform with 2x the amount of weight tolerated by the non-dominant arm b. Have the patient perform ER is standing while using a towel roll to keep the humerus slightly abducted c. If the patient is able to perform 1.5x the maximal volitional isometric contraction of the non-dominant arm in this position, they should be able to return to sport without requiring a progression of this exercise d. Advance the patient to performing standing ER with manual resistance for concentric and eccentric activation while progressing to 90 degrees of abduction

The correct answer is D. Options A and C describe arbitrary benchmarks of strength without ensuring that the patient develops eccentric control of internal rotation in the range required for participation in sport. Option B would be less challenging for the patient's external rotators, as they would not be working against gravity in this position; this exercise would be a greater challenge for balance and stability, but is ultimately not functional enough to assist with return to sport. Option D progresses the patient into a more sport-specific ROM while highlighting eccentric control of internal rotation needed after ball release.

You perform a run analysis on a patient with B knee pain and notice excessive "bounce" in her run. Which of the following is the least likely cause of this excessive vertical displacement? a. Posterolateral hip weakness b. Hip flexor tightness c. Gastroc/soleus tightness d. Restricted talocrural mobility

The correct answer is a. Choices b,c, and d could all be possible causes of increased vertical displacement with running. Posterolateral hip weakness would more likely lead to things like crossover, knee valgus, pronation, etc.

3. Which of the following is NOT a goal of the early post-operative phase (weeks 0-6) after a meniscal repair? a. Avoid any physical touch or manual mobility of the patella b. Restore extension ROM c. Minimize muscular atrophy d. Protect the repair

The correct answer is a. It is important to do manual work on the knee and keep the patella moving in the early post-operative phase. If the therapist does not touch the knee and work on mobility at this time the anterior interval could scar down preventing full extension.

3. Your patient is a R handed volleyball player (often required to serve and spike the ball overhead) presenting with R shoulder pain and weakness. What would be the most appropriate progression of ER strengthening? a. Sidelying ER → Standing ER at 90 degrees ABD b. Sidelying ER with 2 lb weight → Sidelying ER with 4 lb weight c. Standing ER at 90 degrees ABD → Sidelying ER d. Sidelying ER x10 reps → Sidelying ER x20 reps

The correct answer is a. This patient needs strength and power in an overhead position so it would be best to progress from an easier exercise like ER in sidelying to ER in standing ABD which is a movement she will need for her sport. Answers b and d only incorporate ER in sidelying, and answer c starts with a more difficult and more functional exercise and regresses.

1. A patient comes to physical therapy after an ACL reconstruction. The following criteria are met: Full, pain free ROM, active hyperextension with superior patellar glide, 0/10 pain and no joint effusion. This indicates that: a. The patient is safe to return to sport b. The patient is not yet safe to return to sport c. The patient is safe to participate in return to sport testing d. The is not yet safe to participate in return to sport testing

The correct answer is c. Clearance for RTS testing includes full, pain-free ROM, active hyperextension with superior patellar glide, 0-1/10 knee pain, and trace or less joint effusion. These do not indicate that the patient may actually return to sport, but that they are clear to participate in RTS testing.

If a patient presents with shin pain with running and pain on palpation over the distal 2/3 of the posteromedial tibia, what might be a likely cause? a. Achilles tendinopathy b. Plantar fasciopathy c. Medial tibial stress syndrome d. Sural neuropathy

The correct answer is c. These symptoms are common with medial tibial stress syndrome. Achilles tendinopathy would lead to palpation pain more localized to the achilles tendon. Plantar fasciopathy pain would be more in the foot and medial calcaneal tubercle. Sural neuropathy would be more likely to cause symptoms in the foot or more laterally.

2. Which of the following types of meniscal injury are the most ideal for uncomplicated healing/rehab? a. Vertical tears b. Longitudinal tears c. A and B d. None of the above

The correct answer is c. Vertical and longitudinal tears are oriented in line with the meniscal fibers so they are more "ideal" tears than radial, horizontal, or oblique.

Which of the following is a risk factor for patellofemoral pain syndrome? a. Age <30 years b. Male gender c. Hip IR weakness d. Increased knee flexion at midstance

The correct answer is d- increased knee flexion at midstance increases the risk of PFPS. Other risk factors include age >34, female gender, and hip ER weakness which are the opposite of answers a-c.

Patient is a 17 y/o soccer playing who has been an active participant in physical therapy services following ACL reconstruction six months ago. At this time, patient presents with 1/10 pain, trace effusion in the surgical knee, pain only at end range of flexion, and successful active hyperextension with superior patellar glide. Which of the following statements is correct? a. At this time, return to sport testing is not indicated, as the patient has not met the clearance criteria. b. At this time, the patient is ready to return to sport, as the patient presents with minimal pain and joint effusion. c. At this time, return to sport testing may be initiated in order to determine if the patient is ready to return to sport. d. At this time, patient is ready to return to sport, as it has been six months since surgery.

The correct answer is option a. At this time, the patient has not met the clearance criteria for return to sport testing (pain 0-1/10 in the knee, trace or less joint effusion, full/pain-free ROM, and active hyperextension with superior patellar glide); option c is not correct. Options b and d are not correct, as there are many factors such as strength, motor control, agility, and psychological readiness that affect return to sport.

Which of the following interventions may be indicated in the treatment of a patient with midsubstance Achilles tendinosis when the patient presents with excessive pronation during late midstance? a. Eccentrics into DF b. Strengthening of the tibialis posterior c. Eccentrics into neutral only d. A and B are correct e. B and C are correct

The correct answer to this question is option D; both eccentric exercises into DF and strengthening of the tibilias posterior may be indicated for a patient with midsubstance Achilles tendinosis. If the tibialis posterior is unable to eccentrically control foot pronation, the Achilles tendon may compensate, as it attaches medial to the subtalar joint. Therefore, strengthening the Achilles tendon in isolation may not fully resolve this patient's impairments; the tibialis posterior needs to be addressed. Eccentrics into neutral only are indicated for a patient with insertional Achilles tendinosis.

Which of the following exercise interventions is most appropriate for a patient in the advanced stages of rehabilitation following an ACL reconstruction given that the patient is an alpine skier? a. Have the patient perform quad sets progressing to straight leg raises. b. Have the patient stand on two parallel foam rollers while practicing lateral weight shifts in a modified tuck position. c. Have the patient kick into a weighted medicine ball for isometric activation of the quadriceps. d. Have the patient perform a single-leg stand on foam on the surgical side while catching/throwing a weighted ball.

The correct answer to this question is option b, as it is the most sport-specific advanced strengthening exercise. Options a and c both focus on isometric quadriceps activation, which is beneficial during early/intermediate stages of rehabilitation. Option c describes an exercise that is soccer-specific. While option d is an appropriate exercise for this stage of strengthening, it does not incorporate the sport-specific elements (such as the tuck position and the lateral weight shifting mimicking centrifugal forces) that option b does.

Your patient is a collegiate female cross-country runner who presents to your clinic with anterior knee pain. Which of the following is not a potential contributing factor to the development of her patellofemoral pain? a. Increased hip internal rotation in jump-landing task b. Increased femoral inclination c. Increased hip abductor strength d. Increased navicular drop

The correct response is option C; increased hip abductor strength is not a potential contributing factor to the development of PFPS. If a patient has decreased hip abductor strength, they are at increased risk of developing PFPS. Increased hip internal rotation in jump-landing tasks, increased femoral inclination, and increased navicular drop are all risk factors for the development of PFPS.

A patient with restriction in the sagittal plane will most likely present with which of the following abnormal running classifications? a. Over-compliance b. Excessive vertical displacement c. Crossover d. A patient with restriction in the sagittal plane will be able to run without significant change in pattern.

The correct response to this question is option B, excessive vertical displacement. Over-compliance is most often seen due to posterolateral hip weakness, excessive foot pronation, and poor motor control, though decreased dorsiflexion range of motion may contribute to the development of this pattern. Crossover is most commonly seen due to posterolateral hip weakness and may be caused by adductor tightness. Excessive vertical displacement may be caused by hip flexor tightness, gastroc/soleus tightness, or restricted talocrural joint mobility, all of which are restrictions in the sagittal plane. Therefore, option C is correct and option D is incorrect.

Which of the following precautions is not consistent with the rehabilitation of a patient following meniscal root repair? a. No isolated hamstring activation for 6 weeks b. 0-90 degrees of flexion ROM for 2 weeks c. Avoid active/passive hyperextension x6 weeks d. Partial weight bearing 0-2 weeks

The correct response to this question is option d, which describes a weight-bearing status that is more appropriate for a patient with a peripheral longitudinal/vertical tear. Patients should remain non-weight bearing for 6 weeks after mensical root repair. Options a, b, and c all reflect appropriate precautions for patients after mensical root repair.

Your patient is complaining about not being able to reach behind her back to tuck her shirt in. What interventions would be appropriate to help her be able to do this? a. Sleeper stretch b. Anterior GH glide c. Posterior GH glide d. Strengthen shoulder flexors E. Both A & C

There are a few reasons that the patient may be limited in internal rotation. One may be that the posterior capsule is tight. Both the sleeper stretch (A) and posterior GH glides (C) will loosen the capsule and help allow for more movement in the shoulder. Answer B would be helpful if the patient was limited in external rotation or extension. Answer D would apply if the patient was having trouble reaching overhead. Answer E is the correct answer.

A high school baseball pitcher presents to PT with shoulder pain and history of several shoulder subluxations. Upon measurement, he is able to reach 125 degrees of ER in 90 degrees of ABD, but IR in 90 degrees of ABD is limited to 55 degrees. Which of the following would be LEAST appropriate moving forward: a. Upper extremity functional testing b. Stretching the external rotators to increase IR c. Dynamic stabilization exercises d. Test for sulcus sign

With a history of subluxation this patient likely has instability somewhere in the shoulder, so further testing of stability would be appropriate. Therefore answers a and d are incorrect. Dynamic stabilization exercises could be helpful depending on the ultimate diagnosis, so c is not the best answer. The correct answer is b. Many overhead athletes have increased ER and decreased IR, but the total arc of motion is appropriate. Increasing IR would not be beneficial because the patient has an appropriate total arc of motion between ER and IR. Increasing IR would likely decrease the stability of the shoulder and could lead to further complications.

1. Patient presents c/o R shoulder pain. Gross exam of GH rhythm and posture shows R scapula protraction and anterior tilt. MMT will likely confirm which of the following? a. Serratus anterior and lower trapezius 3/5 b. Lower trapezius 5/5 and infraspinatus 3/5 c. Anterior deltoid 3/5 and infraspinatus 5/5 d. Serratus anterior and upper trapezius 5/5

a

A 50 y/o females patient comes in for an eval and treat for R foot pain. She complains that her foot hurts most in the morning when she first steps out of bed in the morning and standing on it after sitting at her desk for a couple of hours. Patient reports that she runs recreationally throughout the week to keep in shape. You note that she has high arches and her DF ROM of her R foot is limited. Upon palpation, you find an area of tenderness around the medial calcaneal tubercle. Squeeze test at the calcaneus is negative. Patient denies numbness/tingling. Which of the following diagnoses more specifically encompasses this patient's symptoms? A) Plantar fasciopathy B) Calcaneal stress fracture C) Fat pad atrophy D) Tarsal tunnel syndrome

a

A patient who is an overhead athlete comes to PT for treatment of his shoulder injury. He has been coming for several weeks and now has minimal pain, full ROM, and symmetrical capsular mobility. What are some exercises or techniques that would be appropriate for this patient who is in this phase of rehabilitation? a. Plyometric exercises such as wall dribbles with two hands initially b. Weight shifting in quadruped position c. Multi angle submaximal isometrics d. PROM and AAROM using wand exercises

a

How do you determine when a patient is ready to begin an interval jogging program? a. Full ROM, </= Trace effusion in knee, pain-free gait, 80% quadriceps symmetry b. Full ROM, pain-free gait, 80% quadriceps symmetry, able to hold 30 second squat at 60 degrees knee flexion c. </= Trace effusion in knee, pain-free gait, 70% quadriceps symmetry, able to hold 30 second squat at 60 degrees knee flexion d. </= Trace effusion in knee, pain-free gait, 70% quadriceps symmetry, 15 minutes walking endurance

a

You watch your patient run on the treadmill and note that he exhibits a crossover pattern when running. Which of the following muscles is most likely the cause of this crossover due to a lack of eccentric control? A) Glute Med B) Glute Max C) Adductor Magnus D) Semitendinosus

a

phases of block periodization

accumulation, transmutation, realization

you evaluated a right-handed 17 y/o male high school baseball pitcher at your outpatient PT clinic. His main complaints include pain in the late cocking phase of throwing, as well as decreased pitching velocity. You suspect anterior instability is the cause. The most appropriate intervention to include in an exercise program for this patient will likely be: Improve IR ROM on his throwing arm Anterior glenohumeral joint mobilizations grade 3 Address deficits in dynamic stability and scapular strength Doorway pectoralis major stretch performed by stepping into doorway

answer C: addressing deficits in dynamic stability and scapular strength, the correct answer. Most pitchers with anterior instability lack stability throughout the necessary ROM needed for pitching, and have impaired force couple relationships needed to keep the shoulder stable during movement (ex. Weak RTC/strong deltoid, serratus anterior/low trap).

How is shoulder ROM (ER/IR) typically affected in the overhead athlete? a. Decreased ER, Increased IR b. Increased ER, Decreased IR c. Increased ER, Increased IR d. Decreased ER, Decreased IR

b

Which of the following post-surgical precautions is specific to a meniscus root repair? TTWB x 2 weeks NWB x 6 weeks ROM flexion restrictions 0-90 degrees for weeks 0-2 PWB x 2 weeks

b

You are having a patient perform the exercise below, wall circles with ball. He is to perform 2 sets of 30 in both clockwise and counterclockwise directions. Which of the following would help to increase rotator cuff muscle activity? a. Have patient perform the exercise with both arms at the same time b. Have patient perform the exercise with an open palm c. Have patient perform the exercise with a flexed elbow d. This exercise does not engage the rotator cuff muscles

b

You are performing a gait and running analysis on a patient, and while running, you notice they exhibit crossover of R > L leg past midline, what is the most likely reason this is happening and how would you proceed with treatment? a. Tight adductor muscles; stretch to restore normal muscle length b. Weak posteriorlateral hip muscles; strengthen to increase stability c. Excessive midfoot pronation; foot intrinsics to improve motor control d. Restricted talocrural mobility; grade 3 - 4 joint mobs to improve mobility

b

You have a 21 year old female patient who had an ACL repair and is eager to return to playing soccer. You want to consider return to sport testing for this patient, and this patient has full, pain free ROM, active hyperextension with superior patellar glide, no knee pain, and little joint effusion. You determine they are cleared for return to sport testing. Which of the following is the correct order in which you should assess return to sport testing for this patient? a. Motor controlàROMàStrengthàSingle leg hopping b. StrengthàMotor controlàSingle leg hoppingàagility c. Single leg hoppingàMotor controlàStrengthàAgility d. StrengthàAgilityàSingle leg hoppingàMotor control

b

you have finished a running assessment for one of your patients, and you notice their step rate is 150 steps per minute. Additionally, you notice she has increased vertical displacement, overstriding, and she complains of anterior shin pain. Would you change this as part of your treatment plan for her, and by how much? a. Yes, it is only appropriate to increase her step rate by 5% (~157) b. Yes, it is appropriate to increase her step rate by 10% (~165) c. No, step rate for this patient should be slower d. No, step rate is appropriate and does not need to be addressed

b

1. Which of the following is NOT a criterion used to clear an athlete for return to sport testing after a knee injury? A. 0-1/10 knee pain B. >4 weeks past the injury date C. Trace/no joint effusion D. Active knee hyperextension with superior glide of patella

b. Correct: It is not required to be more than 4 weeks since the injury date before initiating return to sport testing. Although at 4 weeks most of the other criteria may be met, time is not the most important factor when clearing an athlete to begin return to sport testing.

2. Select the exercise which will most specifically strengthen the supraspinatus: a. Scaption with maximal internal rotation b. Scaption in 'full can' position c. Shoulder abduction with 5 lb weight d. Maintain 'push up plus' position for 30 seconds

b. Correct: performing scaption in the full can position will minimize deltoid activation and more specifically target supraspinatus.

A new patient comes in for an evaluation of his ankle. He is a 40 y/o recreational distance runner of about 30 miles per week, and complains of pain by the back of his ankle, that is worsened when he goes on longer runs. In your examination, you find increased fluid in the middle third of his Achilles tendon, pain with resisted PF, limited ankle DF, hypomobile subtalar joint, and compensated forefoot varus. What would your diagnosis be, and what would you focus on in treatment? a. Achilles Tendonitis; icing painful area, stopping running, ankle stretching b. Achilles Tendonitis; decreasing running intensity, gastroc and soleus heel raises to neutral DF, subtalar joint mobilizations c. Achilles Tendonosis; gastroc and soleus heel raises and lowers to neutral DF, subtalar joint mobilizations, tibialis posterior eccentrics d. Achilles Tendonosis; decreasing running intensity, changing footwear, intrinsic foot muscle strengthening

c

The imbalances in the shoulder capsule in ER/IR may be due to which of the following? a. Posterior capsule laxity and anterior capsule tightness b. Increased humeral anteversion c. Anterior capsule laxity and posterior capsule tightness d. Both anterior and posterior capsule laxity

c

you have a patient who had a two-tunnel transtibial root repair done two weeks ago and are coming to you for physical therapy. Which of the following is NOT A key difference between regular meniscus injury and meniscus root tear? a. Strictly NWB for 6 weeks b. 0-90 ROM restrictions for 2 weeks c. No isolated glute med activation for 3 weeks d. Avoid active/passive hyperextension for 6 weeks

c, no isolated glute med activation for 3 weeks This is NOT a key difference between regular meniscus injury and meniscus root tear because there are no restrictions on isolated glute med activation. However, there is a restriction on isolated hamstring activation for 6 weeks for post-op meniscus root tear. Answer A is incorrect because according to Mueller et al JOSPT 2016, meniscus root tear patients must remain NWB for 6 weeks. According to this same article, meniscal root tear patients must have a 0-90 ROM restriction for 2 weeks, so answer B is also incorrect. Lastly, answer D is incorrect because meniscal root repair patients should avoid active/passive hyperextension for 6 weeks.

1. A 17 y/o male baseball player presents to your outpatient clinic with complaints of R shoulder pain. He reports he has played shortstop for his high school team the past 3 years, and is R hand dominant. Upon examination, you take the following PROM measurements at 90 degrees shoulder abduction: R shoulder IR: 82 R shoulder ER: 108 L shoulder IR: 90 L shoulder ER: 98 The most appropriate course of action is: a. Perform R shoulder internal rotation AAROM to increase IR to 90 degrees. b. Recheck your measurements because these values don't make sense c. Check the patient's dynamic shoulder stability to determine if they can control this entire arc of motion d. Instruct the patient in performing a sleeper stretch on the R

c. Correct: While this patient's ROM values are expected for an overhead athlete, it is important to be sure he has control through this entire motion. If he has instability and loses control at end range for example, this is likely contributing to his pain.

2. A patient presents to your clinical s/p meniscal root repair. It is now 5 weeks since the date of the surgery. The patient is likely cleared for all of the following EXCEPT: A. Knee flexion PROM to 90 degrees B. Soft tissue mobilization to decrease effusion and scar tissue adherence C. Gradually weight shift onto involved leg in standing D. Perform active straight leg raises

c. Correct: the patient must be non-weightbearing for the first 6 weeks to prevent axial load on the repair to encourage full healing.

what is linear periodization

change vol and load predictably, distinct phases so athlete peaks

You are working with a soccer player who is close to return to sport, how would you tailor your treatment to best mimic those conditions? a. Training for endurance with minimal rest breaks b. Eccentric hamstring and concentric quadriceps strength training c. Multiplanar motions with cutting and quick direction changes d. All of the above

d

3. Which of the following is FALSE regarding closed kinetic chain shoulder strengthening? a. CKC strengthening effectively targets the rotator cuff muscles, which can be hard to specifically activate in open chain activities b. CKC strengthening can increase proprioception c. CKC can protect healing tissues in an injured athlete d. CKC should be used with gymnasts, but won't have much benefit for overhead athletes

d. Correct: This statement is FALSE. Overhead athletes will benefit from CKC strengthening by improving stability, neuromuscular control, and will increase throwing velocity, strength and power.

1. After a meniscal repair, what are the goals in the early (0-6 weeks) postoperative phase of rehab? a. Patellar soft tissue mobilization b. Minimize lower extremity atrophy c. Full, pain-free ROM d. All of the above e. A and B

e

what is block periodization

highly concentrated, specialized workloads. doesnt focus on components of training athlete doesnt need

1. What are some benefits to using UE functional testing in closed kinetic chain? a. excellent test re-test and intra-rater reliability has been established b. minimal time and space required c. closed kinetic chain is not reflective of most sport environments d. a and b

rationale: D is correct because there is excellent intra-rater and inter-rater, and test re-test reliability which helps if another therapist has to do a re-assessment on the patient, and it can also help to accurately observe improvements the pt is making and see how effective your treatment is. These tests typically require minimal space and equipment making them easy to use.

stress fx pain vs medial tibial stress syndrome

stress fx: horiz, mtss: vertical

You are treating a young pitcher who is in the advanced stages of PT following a shoulder injury. Before you can safely clear him for his return to pitching, you would like him to meet these milestones: Full ROM, strength, and endurance Adequate proprioception/joint position sense Dynamic stabilization throughout the whole motion of a pitch Minimal to no pain or tenderness All of the above

thrower must have all of the components listed above to be considered for returning to sports, so E is the correct answer. Only addressing ROM, strength, and endurance without the other components will put him at risk of re-injury, since proprioception is often altered following an injury. If a patient has any pain during activities that are not as strenuous as throwing, he will likely only have increased pain with that activity. Pain would also not be common at that point in the rehab process, and would generally mean doing too much or putting the shoulder into a compromising position. Not addressing dynamic stabilization would also put the patient at risk of re-injury, and needs to be thoroughly addressed before beginning a throwing program.

what is nonlinear/undulating periodization

volume load altered more frequently, more recovery time, as lighter loads are used more often. more feq changes in stimuli


Ensembles d'études connexes

anti money-laundering for insurance 2nd edition

View Set

Respiratory System A&P II Part 2

View Set

Chapter 4: Infant Perception and Cognition

View Set

The tragedy of Romeo and Juliet Act III

View Set

chapter 26: Birth-Related Procedures

View Set

Element, Compound, Solution, or Heterogeneous Mixture?

View Set